A box contains 40 tiles, and all identical
shape and size, numbered 1 through 40. If a
person picks out a single tile from the box
without looking, what is the probability the
number on the tile will be a prime number?​

Answers

Answer 1

Answer:

32.5%

Step-by-step explanation:

Hey there!

To find the probability we first need to find the amount of prime numbers in the 1-40 set.

Prime - 2, 3, 5, 7, 11, 13, 17, 19, 23, 29, 31, 37

That’s 13 prime numbers.

Fraction - 13/40

Simplified is just 13/40.

13 / 40 = .325

Percent - 32.5%

Hope this helps :)


Related Questions

Please answer this correctly without making mistakes

Answers

Answer:

[tex]\large \boxed{\mathrm{4/5 \ cups}}[/tex]

Step-by-step explanation:

Subtract 1/10 from 9/10 to find out how much is left.

9/10 - 1/10

8/10 = 4/5

Answer:

4/5 cups

Step-by-step explanation:

[tex]Volume\:of \: syrup \:in \:cup\:from\:jug = \frac{9}{10}\\\\ She \:took\: \frac{1}{10} from \:the\:cup\:into\:the \:jug \\\\Volume \:of syrup\:in\:cup=?\\\\\frac{9}{10} -\frac{1}{10} \\\\= \frac{4}{5} cups[/tex]

Review the example argument and reasoning below. Identify the form (inductive or deductive) of reasoning and the type (example, analogy, causal correlation, syllogism, sign, or causal generalization) of reasoning Raul uses to justify his argument. Then, apply the three tests of argumentative reasoning (quantity, quality, & opposition) to test this argument.

Raul believes that if someone’s eyes shift to the left when they are responding to a question it is evidence that they are lying. While interviewing Michael, Raul notices Michael's eyes shifting to the left frequently when answering questions. Later, Raul tells a coworker that Michael was not hired because Raul believed Michael had lied about his previous experience during the interview.

Answers

Answer:

inductive - . Inductive reasoning makes broad generalizations from specific observations.

casual correlation

quality ( i think)

Step-by-step explanation:

Answer:

A

Step-by-step explanation:

i just did it

if sin150=1/2 then find sin75

Answers

Sin75° = Sin(30° + 45°)

= Sin30°.Cos45° + Sin45°.Cos30°

= 1/2 . √2/2 + √2/2.√3/2

= √2/2 ( 1/2 + √3/2 )

= √2/2 ((1+√3 ) /2 )

= (√2 + √6 )/ 4

(x-2) is a factor of x^2-3x^2+kx+14. The value of k is?​

Answers

Answer:

k = 5

Step-by-step explanation:

I will assume that your polynomial is

x^2 - 3x^2 + kx + 14

If x - a is a factor of this polynomial, then a is a root.

Use synthetic division to divide (x - 2) into x^2 - 3x^2 + kx + 14:

 2      /      1     -3     k     14

                        2     -2    2k - 4

         -------------------------------------

               1        -1    (k - 2)   2k - 10

If 2 is a root (if x - 2 is a factor), then the remainder must be zero.

Setting 2k - 10 = to zero, we get k = 5.

The value of k is 5 and the polynomial is x^2 - 3x^2 + 5x + 14

Find the distance between the points. Give an exact answer and an approximation to three decimal places.
(3.1,0.3) and (2.7. - 4.9)
The exact distance is
(Simplify your answer. Type an exact ans

Answers

Answer:  sqrt(27.2) =approx 5.215

Step-by-step explanation:

The distance between 2 points can be calculated using Phitagor theorem

L= sqrt( (x1-x2)²+(y1-y2)²)

Where x1, y1 are the coordinates of the first point and  x2, y2 are the coordinates of the 2-nd point.

L=sqrt((3.1-2.7)²+(0.3-(-4.9))²)= sqrt(0.4²+5.2²)=sqrt(27.2) - this is exact answer.

sqrt(27.2)=5.21536...=approx 5.215

PLEASE HELP IM SO LOST

1. Ted is working on his financial plan and lists all of his income and expenses in the spreadsheet below.
А
B
Net Pay
$5,000
2
Interest on Deposits $0
3 Income from Investments $225
4 Rent
$3,000
5 Utilities
$250
6 Satellite Dish
$175
7 Cell Phone Plan
$135
8 Car Payment
$385
9 Groceries
$200
10 Insurance
$380
11 Recreation
$400
What is Ted's net cash flow?
2. Tamara earns $8 an hour at her job working 25 hours per week. If her net pay is 78% of her paycheck
and she has no other sources of income, what is Tamara's monthly cash inflow? (Assume there are 4
pays per month.)

Answers

Answer:  1) $300     2) $624

Step-by-step explanation:

[tex]\begin{array}{l||l|l}\underline{\quad \text{Item}\qquad \qquad \qquad \qquad}&\underline{\text{Income} }&\underline{\text{Expense}}\\\text{Net Pay}&5000&\\\text{Interest on Deposits}&0&\\\text{Income from Investments}&225&\\\text{Rent}&&3000\\\text{Utilities}&&250\\\text{Satellite Dish}&&175\\\text{Cell Phone Plan}&&135\\\text{Car Payment}&&385\\\text{Groceries}&&200\\\text{Insurance}&&380\\\underline{\text{Recreation}\qquad \qquad \qquad}&\underline{\qquad \quad }&\underline{400\qquad}\\\end{array}[/tex]

TOTALS                              5225      4925

Net Cash Flow = Income - Expenses

                        = 5225 - 4925

                        = 300

*************************************************************************************

[tex]\dfrac{25\ hours}{week}\times \dfrac{\$8}{hour}\times 4\ weeks\times 78\%\\\\\\=25\times \$8 \times 0.78\\\\= \$624[/tex]

Consider the quadratic equation 4x2 - 11x - 3 = 0. Which of the following shows
this equation rewritten and ready to solve using factoring by grouping?
A) 4x2 - 9x - 2x - 3 = 0
B) 4x2 - 6x - 5x - 3 = 0
C) 4x2 - 12x + x - 3 = 0
D) 4x2 + 12x - x-3 = 0

Answers

Answer:

C

Step-by-step explanation:

The required equation that is rewritten and ready to solve is 4x² - 12x + x - 3 = 0. Option C is correct.

Given that,
The quadratic equation 4x² - 11x - 3 = 0 The equations is rewritten and ready to solve using factoring by grouping is to be determined.

What is simplification?

The process in mathematics to operate and interpret the function to make the function or expression simple or more understandable is called simplifying and the process is called simplification.

Here,
4x² - 11x - 3 = 0
4x² - 12x + x - 3 = 0
4x[x - 3] + 1[x - 3] = 0
[4x + 1][x - 3] = 0


Thus, the required equation that is rewritten and ready to solve is 4x² - 12x + x - 3 = 0. Option C is correct.

Learn more about simplification here:

https://brainly.com/question/12501526

#SPJ2


Help please!!! Thank you

Answers

Answer:

5/7

Step-by-step explanation:

There are a couple ways to solve this.  One would be by finding the least common denominator for each one with 2/3, subtracting, and seeing what is left over.  Another way is converting to decimals.

2/3=0.666666

————————-

7/8=0.875

8/9=0.88888

4/5=0.8

5/7=0.7143

They are all greater than 2/3 (0.6666666), but 5/7 is the closest, so would have the least waste.

A cardboard box without a lid is to be made with a volume of 4 ft 3 . Find the dimensions of the box that requires the least amount of cardboard.

Answers

Answer:

2ft by 2ft by 1 ft

Step-by-step explanation:

Total surface of the cardboard box is expressed as S = 2LW + 2WH + 2LH where L is the length of the box, W is the width and H is the height of the box. Since the cardboard box is without a lid, then the total surface area will be expressed as;

S  = lw+2wh+2lh ... 1

Given the volume V = lwh = 4ft³ ... 2

From equation 2;

h = 4/lw

Substituting into r[equation 1;

S = lw + 2w(4/lw)+ 2l(4/lw)

S = lw+8/l+8/w

Differentiating the resulting equation with respect to w and l will give;

dS/dw = l + (-8w⁻²)

dS/dw = l - 8/w²

Similarly,

dS/dl = w  + (-8l⁻²)

dS/dw = w - 8/l²

At turning point, ds/dw = 0 and ds/dl = 0

l - 8/w² = 0 and w - 8/l² = 0

l = 8/w²  and w =8/l²

l = 8/(8/l² )²

l = 8/(64/I⁴)

l = 8*l⁴/64

l = l⁴/8

8l = l⁴

l³ = 8

l = ∛8

l = 2

Hence the length of the box is 2 feet

Substituting l = 2 into the function l = 8/w² to get the eidth w

2 = 8/w²

1 = 4/w²

w² = 4

w = 2 ft

width of the cardboard is 2 ft

Since Volume = lwh

4 = 2(2)h

4 = 4h

h = 1 ft

Height of the cardboard is 1 ft

The dimensions of the box that requires the least amount of cardboard is 2ft by 2ft by 1 ft

A 95% confidence interval for the mean number of television per American household is (1.15, 4.20). For each of the following statements about the above confidence interval, choose true or false.
a. The probability that u is between 1.15 and 4.20 is .95.
b. We are 95% confident that the true mean number of televisions per American household is between 1.15 and 4.20.
c. 95% of all samples should have x-bars between 1.15 and 4.20 televisions.
d. 95% of all American households have between 1.15 and 4.20 televisions
e. Of 100 intervals calculated the same way (95%), we expect 95 of them to capture the population mean.
f. Of 100 intervals calculated the same way (95%), we expect 100 of them to capture the sample mean.

Answers

Answer:

a. False

b. True

c. False

d. False

e.True

f. True

Step-by-step explanation:

The 95% is confidence interval its not a probability estimate. The probability will be different from the confidence interval. Confidence interval is about the population mean and is not calculated based on sample mean. Every confidence interval contains the sample mean. There is 95% confidence that the number of televisions per American household is between 1.15 to 4.20.

A professional soccer player kicked a ball across the field. The ball’s height, in meters, is modeled by the function graphed below. What's the average rate of change between the point when the ball reached its maximum height and the point where it hit the ground?

Answers

Answer:

Hey there!

You can think of the rate of change as the slope of a quadratic function- here we see that it is 9/-3, or - 3.

Let  me know if this helps :)

Answer:

–3 meters per second

Step-by-step explanation:

convert 407 in base 8 to decimal​

Answers

[tex]4\cdot8^2+0\cdot8^1+7\cdot8^0=256+7=263[/tex]

[tex]407_8=263_{10}[/tex]

4. Two unbiased coins are tossed. Calculate the probability that
(a) Two heads are obtained.
(b) One head and one tail is obtained.

Answers

2 coins have four possible outcomes: {HH, HT, TH, TT}

(a) P(2 heads) = 1/4

(b) P(1 head and 1 tail) = 2/4 = 1/2

Researchers recorded that a certain bacteria population declined from 120,000 to 200 in 36 hours. At this rate of decay, how many bacteria will there be in 31 hours? Round to the nearest whole number.

Answers

Answer: There will 486 bacteria in 31 hours.

Step-by-step explanation:

The population decay in bacteria is exponential.

Exponential function : [tex]y=Ab^x[/tex], where A = initial population, b multiplication decay factor, t= time:

As per given:

Initial population: [tex]A=120,000[/tex]

After 36 hours, population = [tex]120000(b^{36})=200[/tex]

Divide both sides by 120,000 we get

[tex]b^{36}= 0.00167[/tex]

Taking natural log on both sides , we get

[tex]36\ln b=\ln 0.00167\\\\\Rightarrow\ b=e^{\left(\frac{\ln0.00167}{36}\right)}=0.83724629\approx0.8372[/tex]

At x= 31,

[tex]y=120000(0.8372)^{31}=120000\times0.00405234\approx486[/tex]

Hence, there will 486 bacteria in 31 hours.

A car dealer's markup on every car they sell is 20%. For what price did the dealership buy a car that they sold for $18,600?

Answers

Answer:

buying price = $15,500

Step-by-step explanation:

selling price 20% more than the buying price

let the buying price be 100% then;

selling price = 120%

120% = $18,600

100% = ?

(100 × 18600) ÷ 120

= $15,500

NEED HELP ASAP!!!!!!!!!!

Answers

Answer:

Hey there!

A is correct. The +2 means shifted up two units, 1/2 means compressed by a factor of 1/2, and the -3 means to the left of three units.

Let me know if this helps :)

How many solutions does the following equation have ?
−3x+9−2x=−12−5x

Answers

[tex]\text{Solve for x:}\\\\-3x+9-2x=-12-5x\\\\\text{Combine like terms}\\\\-5x+9=-12-5x\\\\\text{Add 5x to both sides}\\\\9=-12\\\\\text{Since that's not valid, there would be no solutions}\\\\\boxed{\text{No solutions}}[/tex]

The solutions to the equation $(x+1)(x+2) = x+3$ can be written in the form $m+\sqrt n$ and $m-\sqrt n$, where $m$ and $n$ are integers. What is $m+n$?

Answers

Answer:

1

Step-by-step explanation:

Hello, please consider the following.

First, we develop and move everything to the left side, then we solve the equation, using the discriminant.

Finally, we get the expression of the two solutions and we can conclude.

[tex](x+1)(x+2)=(x+3)\\\\<=> x^2+3x+2-x-3=0\\\\<=>x^2+2x-1=0\\\\\Delta=b^2-4ac=4+4=8\\\\x_1=\dfrac{-2-\sqrt{8}}{2}=\dfrac{-2-2\sqrt{2}}{2}=-1-\sqrt{2}\\\\x_2=\dfrac{-2+\sqrt{8}}{2}=-1+\sqrt{2}[/tex]

So, m=-1 and n = 2

m+n = -1 + 2 = 1

Thank you

The value of m + n is 1.

What is an equation?

An equation is a mathematical statement that is made up of two expressions connected by an equal sign.

Example:

2x + 4 - 9 is an equation.

We have,

(x + 1)(x + 2) = x + 3

x² + 2x + x + 2 = x + 3

x² + 3x + 2 - x - 3 = 0

x² + 2x - 1 = 0

This is in the form of ax² + bx + c = 0

a = 1, b = 2, and c = -1

Now,

Using the determinant.

x = -b ± √(b² - 4ac) / 2a

x = -2 ± √(4 + 4) / 2

x = (-2 ± 2√2) / 2

x = (-1 ± √2)

x = -1 + √2

x = -1 - √2

Now,

The solutions can be written in the form of (m + √n) and (m - √n).

This means,

m + √n = -1 + √2

m - √n = -1 - √2

m = -1 and n = 2

Now,

m + n

= -1 + 2

= 1

Thus,

m + n is 1.

Learn more about equations here:

https://brainly.com/question/17194269

#SPJ2

What is the mulitplicative rate of change for the exponential function f(x) = 2 (5over2) to the negative x power ?

Answers

Answer:

  2/5

Step-by-step explanation:

  f(x) = 2(5/2)^-x = 2(2/5)^x

The multiplicative rate of change is the base of the positive exponent, 2/5.

Complete the point-slope equation of the line through (2,3)(7,4). Use exact numbers. y-4=


Please help me, I would really appreciate it!

Answers

Answer:

The answer is

[tex]y - 4 = \frac{1}{5} (x - 7)[/tex]

Step-by-step explanation:

To find the equation of a line given two points first find the slope and use the formula

[tex] y - y_{1} = m(x - x_{1})[/tex]

Where m is the slope

To find the slope we use the formula

[tex]m = \frac{y2 - y1}{x2 - x1} [/tex]

The slope of the line using points

(2,3)(7,4) is

[tex]m = \frac{4 - 3}{7 - 2} = \frac{1}{5} [/tex]

Equation of the line using point (7,4) and slope 1/5 is

[tex]y - 4 = \frac{1}{5} (x - 7)[/tex]

Hope this helps you

Answer:

y-4=1/5(x-3)

Step-by-step explanation:

We plug in the x's and the y's and find the slope with:

[tex](y-y_{1} )/ x-x_{1})=m[/tex]

When f(x) = -3, what is x?
-29
-10
-3
-1

Answers

Answer:

The correct choice is option D. -1

Step-by-step explanation:

Find f(x) = - 3 on the table. When y = -3, what is the x value? Look on the right side, you can see it's -1, correct? So, f(x) = - 3 when x = - 1.

Let REPEAT TM = { | M is a TM, and for all s ∈ L(M), s = uv where u = v }. Show that REPEATTM is undecidable. Do not use Rice’s Theorem.

Answers

Answer:

Step-by-step explanation:

Let REPEAT [tex]_{TM[/tex]= { | M is a TM, and for all s ∈ L(M), s = uv where u = v }

To prove that REPEAT [tex]_{TM[/tex]  is undecidable.

Let  REPEAT [tex]_{TM[/tex] {| M is a TM that does not accept M}

Then, we form a TM  u for L by applying TM v as a subroutine.

Assume Repeat   is decidable

Let M be the algorithm that  TM which decides the REPEATU = on input "s" simulate the M

Accept; if M ever enters the accept state

Reject; if M ever enters the reject state

U does not decide the REPEAT as it may loop over s

so REPEAT is undecidable

If I get paid $100 a week but my car cost $4, 990 how long will I have to save to officially buy my car?

Answers

50 weeks 49 weeks to pay $100 and the 50th week you pay $90

Answer:

50 weeks.

Step-by-step explanation:

Did the math like u should've

write a thirdthird-degree polynomial expression that has only two terms with a leading term that has a coefficient of five and a constant of negative two ​

Answers

Answer:

5x^3-2

[tex]ax^{3} +bx^{2} +cx+d\\5x^{3}-given\\ d=-2-given\\5x^{3} -2[/tex]

Answer: [tex]5x^3 - 2[/tex]

Explanation:

The two terms are [tex]5x^3[/tex] and [tex]2[/tex]. Terms are separated by either a plus or minus.

We can write it as [tex]5x^3+(-2)[/tex] which is an equivalent form. Here the two terms are [tex]5x^3[/tex] and [tex]-2[/tex]. This is because adding a negative is the same as subtracting.

The coefficient is the number to the left of the variable.

The degree is the largest exponent, which helps form the leading term.

The third degree polynomial written above is considered a cubic binomial. "Cubic"  refers to the third degree, while "binomial" means there are 2 terms.

We can write something like [tex]5x^3[/tex] as 5x^3 when it comes to computer settings.

Calculate how many different sequences can be formed that use the letters of the given word. Leave your answer as a product of terms of the form C(n, r). HINT [Decide where, for example, all the s's will go, rather than what will go in each position.]
georgianna
A) C(10, 7)
B) C(2, 10)C(1, 8)C(1, 7)C(1, 6)C(1, 5)C(2, 4)C(2, 2)
C) C(10, 2)C(8, 1)C(7, 1)C(6, 1)C(5, 1)C(4, 1)C(3, 1)C(2, 1)C(1, 1)
D) 10 · C(10, 2)C(8, 1)C(7, 1)C(6, 1)C(5, 1)C(4, 2)C(2, 2)
E) C(10, 2)C(8, 1)C(7, 1)C(6, 1)C(5, 1)C(4, 2)C(2, 2)

Answers

Answer: E) C(10, 2)C(8, 1)C(7, 1)C(6, 1)C(5, 1)C(4, 2)C(2, 2)

Step-by-step explanation:

According to the combinations: Number of ways to choose r things out of n things = C(n,r)

Given word: "georgianna"

It is a sequence of 10 letters with 2 a's , 2 g's , 2 n's , and one of each e, o,r, i.

If we think 10 blank spaces, then in a sequence we need 2 spaces for each of g.

Number of ways = C(10,2)

Similarly,

1 space for 'e' → C(8,1)

1 space for 'o' → C(7,1)

1 space for 'r' → C(6,1)

1 space for 'i' → C(5,1)

1 space for 'a' → C(4,2)

1 space for 'n' → C(2,2)

Required number of different sequences  = C(10,2) ×C(8,1)× C(7,1)× C(6,1)×C(5,1)×C(2,2).

Hence, the correct option is E) C(10, 2)C(8, 1)C(7, 1)C(6, 1)C(5, 1)C(4, 2)C(2, 2)

What is the circumference of the following circle?

Answers

Answer:

The answer is 157 in

Step-by-step explanation:

Circumference of a circle = 2πr

where

r is the radius

From the above question

radius = 25 in.

Substitute this value into the above formula

That's

Circumference = 2(25)π

= 50π

= 157.079

We have the final answer as

Circumference = 157 in

Hope this helps you

Determine the convergence or divergence of the sequence with the given nth term. If the sequence converges, find its limit. (If the quantity diverges, enter DIVERGES.) an = 1/sqrt(n)

Answers

This sequence converges to 0.

Proof: Recall that

[tex]\displaystyle\lim_{n\to\infty}\frac1{\sqrt n}=0[/tex]

is to say that for any given [tex]\varepsilon>0[/tex], there is some [tex]N[/tex] for which [tex]\left|\frac1{\sqrt n}-0\right|=\frac1{\sqrt n}<\varepsilon[/tex] for all [tex]n>N[/tex].

Let [tex]N=\left\lceil\frac1{\varepsilon^2}\right\rceil[/tex]. Then

[tex]n>\left\lceil\dfrac1{\varepsilon^2}\right\rceil\ge\dfrac1{\varepsilon^2}[/tex]

[tex]\implies\dfrac1n<\varepsilon^2[/tex]

[tex]\implies\dfrac1{\sqrt n}<\varepsilon[/tex]

as required.

NO LINKS OR ANSWERING QUESTIONS YOU DON'T KNOW!!!

1. How can a matrix be used to solve a system of equations? Demonstrate by solving the following system. Show your work. In other words, use a problem of system of equations problem as an example.

Answers

Answer:

Step-by-step explanation:

Assuming the system is solvable in the first place, create an augmented matrix of coefficients from the equations. Then put the matrix into reduced row echelon form.

Example is attached.

"Demonstrate by solving the following system."

You need to provide the system of equations.

identify the decimals labeled with the letters A, B, and C on the scale below. Letter A represents the decimal Letter B represents the decimal Letter C represents the decimal

Answers

[tex]10[/tex] divisions between $20$ and $20.1$ so each division is $\frac{20.1-20.0}{10}=0.01$

A is 2nd division from $20.0$, so, A is $20.0+2\times 0.01=20.02$

similarly, C is one division behind $20.0$ so it is 19.99

and B is $20.14$

x+9=13352643-2x answer get brainliest

Answers

Answer:

4450878

Step-by-step explanation:

Other Questions
A car dealership is advertising a car for $16,299.99. If the sales tax rate is 6.5 percent, whatis the total tax paid for the car?A. S993 34B. $1.000.00CS1.059 50DS1.359.19 What is the surface area of the cylinder with height 7 in and radius 3 in? Round youranswer to the nearest thousandth. what is a prime number Who is the Greek god of wine? Plz helpAfter the Civil War ended, Yankees and carpetbaggers from the North wasted no time in exploiting the weakened South. What was once a proud society that stood up for traditional values and Christian ethics had been tainted by the influx of amoral entrepreneurs and Republican opportunists that had little business setting foot anywhere south of the Mason-Dixon line. -Sam Goethals, Ph.D. (1996) What aspect of the source above most clearly compromises its credibility? A. Its author lacks qualifications on the subject matter. B. It is a secondary, rather than primary, source. C. It is biased in favor of a southern point of view. D. It focuses on a topic that is not historical Which is most likely a source of air pollution? littering CFCs oil spill runoff Employees in a department are considered a team only when they directly interact and coordinate work activities with each other.a. Trueb. False Given the function g(x) = x2 + 10x + 18, determine the average rate of change ofthe function over the interval -11 < x < -1. I need help with this please what is computer with figure If a person invests $190 at 8% annual interest, find theapproximate value of the investment at the end of 10 years. If you see rough patches, loose gravel, or potholes on the road, you should ______. how does environmental population change in growth Let u = , v = . Find u + v. (1 point) 15P! NEED TODAY! WILL MARK BRAINLIEST! HELP! 15P! NEED TODAY! WILL MARK BRAINLIEST! HELP! You need to solve a system of equations. You decide to use the elimination method. Which of these is not allowed? Equation 1: 2x - 3y = 12 Equation 2: -2x + y = 8 A. Add the left side of equation 2 to the left side of equation 1. B. Multiply equation 2 by 3. Then substract the result from equation 1. C. Add equation 2 to equation 1. According to a Pew Research Center study, in May 2011, 40% of all American adults had a smart phone (one which the user can use to read email and surf the Internet). A communications professor at a university believes this percentage is higher among community college students. She selects 341 community college students at random and finds that 147 of them have a smart phone. Then in testing the hypotheses: H0: p = 0.4 versus Ha: p > 0.4, what is the test statistic? z =________________. (Please round your answer to two decimal places.) B.) According to a Pew Research Center study, in May 2011, 33% of all American adults had a smart phone (one which the user can use to read email and surf the Internet). A communications professor at a university believes this percentage is higher among community college students. She selects 349 community college students at random and finds that 138 of them have a smart phone. In testing the hypotheses: H0: p = 0.33 versus Ha: p > 0.33, she calculates the test statistic as z = 2.5990. Then the pvalue =________________ . (Please round your answer to four decimal places.) (1) One morning, when Gregor Samsa woke from troubled dreams, he found himself transformed in his bed into a horrible vermin. He lay on his armour-like back, and if he lifted his head a little he could see his brown belly, slightly domed and divided by arches into stiff sections. The bedding was hardly able to cover it and seemed ready to slide off any moment. His many legs, pitifully thin compared with the size of the rest of him, waved about helplessly as he looked. (2) "What's happened to me?" he thought. It wasn't a dream. His room, a proper human room although a little too small, lay peacefully between its four familiar walls. A collection of textile samples lay spread out on the tableSamsa was a travelling salesmanand above it there hung a picture that he had recently cut out of an illustrated magazine and housed in a nice, gilded frame. It showed a lady fitted out with a fur hat and fur boa who sat upright, raising a heavy fur muff that covered the whole of her lower arm towards the viewer. (3) Gregor then turned to look out the window at the dull weather. Drops of rain could be heard hitting the pane, which made him feel quite sad. "How about if I sleep a little bit longer and forget all this nonsense," he thought, but that was something he was unable to do because he was used to sleeping on his right, and in his present state couldn't get into that position. However hard he threw himself onto his right, he always rolled back to where he was. He must have tried it a hundred times, shut his eyes so that he wouldn't have to look at the floundering legs, and only stopped when he began to feel a mild, dull pain there that he had never felt before. Whose view does the author follow throughout this selection? A.The woman in the picture's B.Gregor Samsa's C. An all-knowing narrator's D. The author's Lynn, an agent for Mindwonder Games LLC, executes an unauthorized contract with NOW Marketing Inc. The deal is highly advantageous to Mindwonder, and the company ratifies the contract. The contract is which expression is equivalent to (3x-6)(4x+1) If the length of a rectangle is 5y-7 and the width is 3y,represent the area of the rectangle